Problemas Worpole

August 25, 2017 | Author: Erick García | Category: Statistics, Car, Hypothesis, Sampling (Statistics), Tire
Share Embed Donate


Short Description

Descripción: Ejercicios del capitulo de pruebas de hipótesis resueltos con software estadístico R...

Description

Instituto Politécnico Nacional Unidad Profesional Interdisciplinaria de Ingeniería Campus Guanajuato

Estadística Industrial Ejercicios “Prueba de hipótesis” Arroyo Pozas Vanessa Irisema Profesor: Juan Ignacio Guizar Ruiz. 3IV1

Instrucciones: Realiza los siguientes ejercicios del libro base para el curso. Ejercicios: 10.35, 10.39, 10.40, 10.41, 10.42, 10.43, 10.44, 10.45, 10.53, 10.54, 10.55, 10.56, 10.57, 10.58, 10.59, 10.60, 10.61, 10.62, 10.67, 10.74, 10.75, 10.76, 10.77, 10.78. Los datos que se emplean en estos ejercicios están en la semana 8 del curso. Para la realización de estos ejercicios deberá plantear formalmente las hipótesis correspondientes, indicar el estadístico de prueba a utilizar, obtener el valor del estadístico de prueba para los datos y dar las conclusiones haciendo referencia al nivel de significancia empleado (de preferencia utilizando un valor P). Deberá emplear el software R como apoyo para solucionar los ejercicios

10.35 Para indagar si un nuevo suero frena el desarrollo de la leucemia se seleccionan 9 ratones, todos en una etapa avanzada de la enfermedad. Cinco ratones reciben el tratamiento y cuatro no. Los tiempos de supervivencia, en años, a partir del momento en que comienza el experimento son los siguientes: Con tratamiento Sin tratamiento

2.1

5.3

1.4

4.6

1.9

0.5

2.8

3.1

0.9

A un nivel de significancia de 0.05, ¿se puede decir que el suero es eficaz? Suponga que las dos poblaciones se distribuyen de forma normal con varianzas iguales. Datos Varianzas iguales n=9 Nivel de significancia=0.05 1) Planteamiento H 0 : μ CT =μ ST H 1 : μCT > μ ST

2) Estadístico de prueba T=

( ´xCT − x´ ST )−(μCT −μ ST ) Sp



1 1 + nCT n ST

T t NCT +NST −2 HO

Bajo

( ´x CT −´x ST )

T=

Sp



1 1 + n CT nST

3) Criterio de rechazo (1) Se rechaza

HO

Aquí n-1= 9-2= 7 α =0.05

|T o|=0.669

si

|T o|≥t (n−2) α

(n−2)



=1.894579

|0.669| 10

2) Estadístico de prueba T=

( ´xCT − x´ ST )−(μCT −μ ST ) Sp



1 1 + nCT n ST

T t E 1 +E 2−2 HO

Bajo

( ´x CT −´x ST )

T0=

Sp



1 1 + n CT nST

3) Criterio de rechazo (1) Se rechaza

HO

si

Aquí n-1= 12-2= 10 α =0.1

|T o|=0.1845 t (n−2) =1.372184 α

|0.1845|0

2) Estadístico de prueba T=

( ´x F −´x NF )−(μF −μ NF )

√ HO

Bajo T0=

s2F s 2NF + nF n NF

( ´x F −´x NF )



s 2F s 2NF + n F n NF

3) Criterio de rechazo P-valor=0.6876 Se rechaza si

valor de significancia ≥ 0.6876

No se rechaza si

valor de significancia2.0518 Como

|T o|>t (n−1) α/ 2

se rechaza

3) Criterio de rechazo (2) Se rechaza si

P−valor< α

P-valor=0.01261 α =0.05

Como

0.012610.05

no se rechaza

4) Conclusión No hay evidencia suficiente para concluir que existe una diferencia entre un análisis químico de laboratorio y un análisis de fluorescencia de rayos X del contenido de hierro, por lo tanto no se rechaza H O

10.43 De acuerdo con informes publicados, el ejercicio en condiciones de fatiga altera los mecanismos que determinan el desempeño. Se realizó un experimento con 15 estudiantes universitarios hombres, entrenados para realizar un movimiento horizontal continuo del brazo, de derecha a izquierda, desde un microinterruptor hasta una barrera, golpeando sobre la barrera en coincidencia con la llegada de una manecilla del reloj a la posición de las 6 en punto. Se registró el valor absoluto de la diferencia entre el tiempo, en milisegundos, que toma golpear sobre la barrera y el tiempo para que la manecilla alcance la posición de las 6 en punto (500 mseg). Cada participante ejecutó la tarea cinco veces en condiciones sin fatiga y con fatiga, y se registraron las siguientes sumas de las diferencias absolutas para las cinco ejecuciones:

Un aumento en la diferencia media absoluta de tiempo cuando la tarea se ejecuta en condiciones de fatiga apoyaría la afirmación de que el ejercicio, en condiciones de fatiga, altera el mecanismo que determina el desempeño. Suponga que las poblaciones se distribuyen normalmente y pruebe tal afirmación. Datos Varianzas diferentes Nivel de significancia=0.05

Pareado 1) Planteamiento

H 0 : μ E 1−μ E 2=0 H 1 : μ E 1−μ E 2 ≠0 2) Estadístico de prueba T=

d´ −d 0 s D /√ n

T t n−1 Bajo T0=

HO d´ s D /√ n

3) Criterio de rechazo (1) Se rechaza

HO

|T o|≥t (n−1) α/ 2

si

Aquí n-1= 15-1= 14 α /2=0.025

|T o|=|−2.5259| (n−1)

t α /2 =−2.1447

|−2.5259|>−2.1447 Como

|T o|>t (n−1) α/ 2

se rechaza

3) Criterio de rechazo (2) Se rechaza si

P−valor< α

P-valor=0.02422 α =0.05 Como

0.02422−2.3646 Como

|T o|>t (n−1) α/ 2

se rechaza

3) Criterio de rechazo (2) Se rechaza si

P−valor< α

P-valor=0.03186 α =0.05

Como

0.03186 0.05

no se rechaza

4) Conclusión No hay razón suficiente para dudar de la afirmación del experto en mercadotecnia. No se rechaza H O

10.56 Suponga que, en el pasado, 40% de todos los adultos estaban a favor de la pena capital. ¿Existe alguna razón para creer que la proporción de adultos que está a favor de la pena capital ha aumentado si, en una muestra aleatoria de 15 adultos, 8 están a favor de la pena capital? Utilice un nivel de significancia de 0.05 Datos n=15 x=8 ^p=

8 15

Nivel de significancia de 0.05 Binomial 1) Planteamiento H 0 : p=0.40 H 1 : p>0.40

2) Estadístico de prueba P−valor=p ( X ≥ 8 dado que p=0.40)

15−x

0.60 ¿ 0.40¿ x ¿ 15

¿ ∑ C 15 x ¿ x=8

3)

Criterio rechazo

de (2) Se rechaza si

P−valor< α

P-valor=0.2131 α =0.05

Como

0.2131>0.05

no se rechaza

4) Conclusión No existe evidencia suficiente para concluir que la proporción de adultos que están a favor de la pena capital ha aumentado, dado que tenemos un p-valor>α, por lo tanto no se rechaza H0

10.57 Se está considerando utilizar un nuevo aparato de radar para cierto sistema de misiles de defensa. El sistema se verifica experimentando con una aeronave en la que se simula una situación en la que alguien muere y otra en la que no ocurre ninguna muerte. Si en 300 ensayos ocurren 250 muertes, al nivel de significancia de 0.04, acepte o rechace la afirmación de que la probabilidad de una muerte con el nuevo sistema no excede a la probabilidad de 0.8 del sistema que se utiliza actualmente. Datos n=300 x=¿ 250

^p=

250 300

Nivel de significancia de 0.04

1) Planteamiento H 0 : p=0.8 ↔ p o H 1 : p>0.8 ↔ p o

2) Estadístico de prueba Z=

Bajo



^p− p N ( 0,1) p (1− p) n

HO

Zo=

^p − po



p o (1− p o) n

N (0,1)

3) Criterio de rechazo Se rechaza

HO

si

Z o ≤−Z α

P−valor=p (Z ≤ Z o )

Zo=

0.8333−0.8 0.8 ( 1−0.8 ) 300



Z o =1.441932

P−valor=p ( Z >1.441932 )=0.9253392 Se rechaza si

P−valor< α

P-valor= 0.9253392 α =0.04 Como

0.9253392>0.04

0.5159534 4) Conclusión

no se rechaza

10.58 Se cree que al menos 60% de los residentes de cierta área están a favor de una demanda de anexión de una ciudad vecina. ¿Qué conclusión extraería si sólo 110 en una muestra de 200 votantes están a favor de la demanda? Utilice un nivel de significancia de 0.05. Datos n=200 x=110 ^p=

110 200

1) Planteamiento H 0 : p=0.6 H 1 : p0.93)

4) Conclusión No hay evidencia suficiente para concluir que la nueva medicina es eficaz

10.62 En un experimento de laboratorio controlado, científicos de la Universidad de Minnesota descubrieron que 25% de cierta cepa de ratas sujetas a una dieta con 20% de grano de café y luego forzadas a consumir un poderoso químico causante de cáncer desarrollaron tumores cancerosos. Si el experimento se repite, y 16 de 48 ratas desarrollan tumores, ¿existen razones para creer que la proporción de ratas que desarrollan tumores cuando se someten a esta dieta se incrementa? Utilice un nivel de significancia de 0.05.

10.67 Se sabe que el contenido de los envases de un lubricante específico se distribuye normalmente con una varianza de 0.03 litros. 2 Pruebe la hipótesis de que σ =0.03 contra la alternativa de que 2

σ ≠ 0.03 para la muestra aleatoria de 10 envases del ejercicio 10.23 de la página 356. Use un valor P en sus conclusiones. Datos 2

σ =0.03 n=10 s 2=0.060 1) Planteamiento 2 H 0 : σ =0.03 H 1 : σ 2 ≠ 0.03 2) Estadístico de prueba X 2=

Bajo

( n−1)s 2 2 X (n−1) σ2 HO

(n−1) s 2 X = σ 2o 2 o

2

X o =18.13 Como P−valor=2∙ P(X 2> X 2O ) 3) Criterio de rechazo Se rechaza

HO

Se rechaza si

si

Z o ≤−Z α

P−valor< α

P-valor=0.04643 α =0.05

10.74 En el ejercicio 10.41 de la página 358 pruebe la hipótesis a un nivel de significancia de 0.05 de que σ1 2 = σ2 2 contra la alternativa de que σ1 2 ≠ σ2 2, donde σ1 2 y σ2 2 son las varianzas para el número de organismos por metro cuadrado de agua en los dos lugares diferentes de Cedar Run. 10.75 Remítase al ejercicio 10.39 de la página 358 y pruebe la hipótesis de que σ1 2 = σ2 2 contra la alternativa de que σ1 2 ≠ σ2 2, donde σ1 2 y σ2 2 son las varianzas para la duración de las películas producidas por la empresa 1 y la empresa 2, respectivamente. Utilice un valor P. 10.76 Se comparan dos tipos de instrumentos para medir la cantidad de monóxido de azufre en la atmósfera en un experimento sobre la contaminación del aire. Los investigadores desean determinar si los dos tipos de instrumentos proporcionan mediciones con la misma variabilidad. Se registran las siguientes lecturas para los dos instrumentos:

Suponga que las poblaciones de mediciones se distribuyen de forma aproximadamente normal y pruebe la hipótesis de que σA = σB, contra la alternativa de que σA ≠ σB. Use un valor P. 10.77 Se lleva a cabo un experimento para comparar el contenido de alcohol en una salsa de soya en dos líneas de producción diferentes. La producción se supervisa ocho veces al día. A continuación se presentan los datos. Línea de producción 1. 0.48 0.39 0.42 0.52 0.40 0.48 0.52 0.52 Línea de producción 2. 0.38 0.37 0.39 0.41 0.38 0.39 0.40 0.39 Suponga que ambas poblaciones son normales. Se sospecha que la línea de producción 1 no está produciendo tan consistentemente como la línea 2 en términos de contenido de alcohol. Pruebe la hipótesis de que σ1 = σ2 contra la alternativa de que σ1 ≠ σ2. Utilice un valor P 10.78 Se sabe que las emisiones de hidrocarburos de los automóviles disminuyeron de forma drástica durante la década de 1980. Se realizó un estudio para comparar las emisiones de hidrocarburos a velocidad estacionaria, en partes por millón (ppm), para automóviles de 1980 y 1990. Se seleccionaron al azar 20 automóviles de cada modelo y se registraron sus niveles de emisión de hidrocarburos. Los datos son los siguientes: Modelos 1980: 141 359 247 940 882 494 306 210 105 880 200 223 188 940 241 190 300 435 241 380 Modelos 1990: 140 160 20 20 22360 20 95 360 70 220 400 217 58 235 380 200 175 85 65 Pruebe la hipótesis de que σ1 = σ2 contra la alternativa de que σ1 ≠ σ2. Suponga que ambas poblaciones son normales. Utilice un valor P.

View more...

Comments

Copyright ©2017 KUPDF Inc.
SUPPORT KUPDF